Sum of two co-prime integers











up vote
2
down vote

favorite












I need some help in a proof:
Prove that for any integer $n>6$ can be written as a sum of two co-prime integers $a,b$ s.t. $gcd(a,b)=1$.



I tried to go around with "Dirichlet's theorem on arithmetic progressions" but didn't had any luck to come to an actual proof.
I mainly used arithmetic progression of $4$, $(4n,4n+1,4n+2,4n+3)$, but got not much, only to the extent of specific examples and even than sometimes $a,b$ weren't always co-prime (and $n$ was also playing a role so it wasn't $a+b$ it was $an+b$).



I would appriciate it a lot if someone could give a hand here.










share|cite|improve this question









New contributor




Daniel Gimpelman is a new contributor to this site. Take care in asking for clarification, commenting, and answering.
Check out our Code of Conduct.
















  • 4




    What's wrong with $n=1+(n-1)$? Perhaps you meant to require $a,b>1$ but you should specify that.
    – lulu
    5 hours ago








  • 1




    Dirichlet’s theorem is useless in that kind of situation; the problem is much, much simpler. Aside from the obvious case $n=(n-1)+1$, you can try to find general patterns when $n=4m$, $n=2m+1$, $n=4m+2$.
    – Mindlack
    5 hours ago






  • 1




    True i should have specified that a,b>1, sorry, my bad.
    – Daniel Gimpelman
    5 hours ago










  • Daniel, you can edit your post. Also, have a look at this introduction to posting mathematical notation.
    – hardmath
    5 hours ago










  • Mindlack thanks, but I don't really understand how to go from here. How do I represent a and b even in a specific case? $7=2*3+1$ where $a=2$ and $b=1$? $8=4*2$ where $a=4$ and $b=0$? (And than b isn't co-prime). $10=4*2+2$ where $a=4$ and $b=2$ (a,b aren't co-prime). Also there would always be the m, which i'm not sure if the m could be greater than 1.
    – Daniel Gimpelman
    4 hours ago

















up vote
2
down vote

favorite












I need some help in a proof:
Prove that for any integer $n>6$ can be written as a sum of two co-prime integers $a,b$ s.t. $gcd(a,b)=1$.



I tried to go around with "Dirichlet's theorem on arithmetic progressions" but didn't had any luck to come to an actual proof.
I mainly used arithmetic progression of $4$, $(4n,4n+1,4n+2,4n+3)$, but got not much, only to the extent of specific examples and even than sometimes $a,b$ weren't always co-prime (and $n$ was also playing a role so it wasn't $a+b$ it was $an+b$).



I would appriciate it a lot if someone could give a hand here.










share|cite|improve this question









New contributor




Daniel Gimpelman is a new contributor to this site. Take care in asking for clarification, commenting, and answering.
Check out our Code of Conduct.
















  • 4




    What's wrong with $n=1+(n-1)$? Perhaps you meant to require $a,b>1$ but you should specify that.
    – lulu
    5 hours ago








  • 1




    Dirichlet’s theorem is useless in that kind of situation; the problem is much, much simpler. Aside from the obvious case $n=(n-1)+1$, you can try to find general patterns when $n=4m$, $n=2m+1$, $n=4m+2$.
    – Mindlack
    5 hours ago






  • 1




    True i should have specified that a,b>1, sorry, my bad.
    – Daniel Gimpelman
    5 hours ago










  • Daniel, you can edit your post. Also, have a look at this introduction to posting mathematical notation.
    – hardmath
    5 hours ago










  • Mindlack thanks, but I don't really understand how to go from here. How do I represent a and b even in a specific case? $7=2*3+1$ where $a=2$ and $b=1$? $8=4*2$ where $a=4$ and $b=0$? (And than b isn't co-prime). $10=4*2+2$ where $a=4$ and $b=2$ (a,b aren't co-prime). Also there would always be the m, which i'm not sure if the m could be greater than 1.
    – Daniel Gimpelman
    4 hours ago















up vote
2
down vote

favorite









up vote
2
down vote

favorite











I need some help in a proof:
Prove that for any integer $n>6$ can be written as a sum of two co-prime integers $a,b$ s.t. $gcd(a,b)=1$.



I tried to go around with "Dirichlet's theorem on arithmetic progressions" but didn't had any luck to come to an actual proof.
I mainly used arithmetic progression of $4$, $(4n,4n+1,4n+2,4n+3)$, but got not much, only to the extent of specific examples and even than sometimes $a,b$ weren't always co-prime (and $n$ was also playing a role so it wasn't $a+b$ it was $an+b$).



I would appriciate it a lot if someone could give a hand here.










share|cite|improve this question









New contributor




Daniel Gimpelman is a new contributor to this site. Take care in asking for clarification, commenting, and answering.
Check out our Code of Conduct.











I need some help in a proof:
Prove that for any integer $n>6$ can be written as a sum of two co-prime integers $a,b$ s.t. $gcd(a,b)=1$.



I tried to go around with "Dirichlet's theorem on arithmetic progressions" but didn't had any luck to come to an actual proof.
I mainly used arithmetic progression of $4$, $(4n,4n+1,4n+2,4n+3)$, but got not much, only to the extent of specific examples and even than sometimes $a,b$ weren't always co-prime (and $n$ was also playing a role so it wasn't $a+b$ it was $an+b$).



I would appriciate it a lot if someone could give a hand here.







number-theory elementary-number-theory






share|cite|improve this question









New contributor




Daniel Gimpelman is a new contributor to this site. Take care in asking for clarification, commenting, and answering.
Check out our Code of Conduct.











share|cite|improve this question









New contributor




Daniel Gimpelman is a new contributor to this site. Take care in asking for clarification, commenting, and answering.
Check out our Code of Conduct.









share|cite|improve this question




share|cite|improve this question








edited 4 hours ago









Avi Steiner

2,693927




2,693927






New contributor




Daniel Gimpelman is a new contributor to this site. Take care in asking for clarification, commenting, and answering.
Check out our Code of Conduct.









asked 5 hours ago









Daniel Gimpelman

112




112




New contributor




Daniel Gimpelman is a new contributor to this site. Take care in asking for clarification, commenting, and answering.
Check out our Code of Conduct.





New contributor





Daniel Gimpelman is a new contributor to this site. Take care in asking for clarification, commenting, and answering.
Check out our Code of Conduct.






Daniel Gimpelman is a new contributor to this site. Take care in asking for clarification, commenting, and answering.
Check out our Code of Conduct.








  • 4




    What's wrong with $n=1+(n-1)$? Perhaps you meant to require $a,b>1$ but you should specify that.
    – lulu
    5 hours ago








  • 1




    Dirichlet’s theorem is useless in that kind of situation; the problem is much, much simpler. Aside from the obvious case $n=(n-1)+1$, you can try to find general patterns when $n=4m$, $n=2m+1$, $n=4m+2$.
    – Mindlack
    5 hours ago






  • 1




    True i should have specified that a,b>1, sorry, my bad.
    – Daniel Gimpelman
    5 hours ago










  • Daniel, you can edit your post. Also, have a look at this introduction to posting mathematical notation.
    – hardmath
    5 hours ago










  • Mindlack thanks, but I don't really understand how to go from here. How do I represent a and b even in a specific case? $7=2*3+1$ where $a=2$ and $b=1$? $8=4*2$ where $a=4$ and $b=0$? (And than b isn't co-prime). $10=4*2+2$ where $a=4$ and $b=2$ (a,b aren't co-prime). Also there would always be the m, which i'm not sure if the m could be greater than 1.
    – Daniel Gimpelman
    4 hours ago
















  • 4




    What's wrong with $n=1+(n-1)$? Perhaps you meant to require $a,b>1$ but you should specify that.
    – lulu
    5 hours ago








  • 1




    Dirichlet’s theorem is useless in that kind of situation; the problem is much, much simpler. Aside from the obvious case $n=(n-1)+1$, you can try to find general patterns when $n=4m$, $n=2m+1$, $n=4m+2$.
    – Mindlack
    5 hours ago






  • 1




    True i should have specified that a,b>1, sorry, my bad.
    – Daniel Gimpelman
    5 hours ago










  • Daniel, you can edit your post. Also, have a look at this introduction to posting mathematical notation.
    – hardmath
    5 hours ago










  • Mindlack thanks, but I don't really understand how to go from here. How do I represent a and b even in a specific case? $7=2*3+1$ where $a=2$ and $b=1$? $8=4*2$ where $a=4$ and $b=0$? (And than b isn't co-prime). $10=4*2+2$ where $a=4$ and $b=2$ (a,b aren't co-prime). Also there would always be the m, which i'm not sure if the m could be greater than 1.
    – Daniel Gimpelman
    4 hours ago










4




4




What's wrong with $n=1+(n-1)$? Perhaps you meant to require $a,b>1$ but you should specify that.
– lulu
5 hours ago






What's wrong with $n=1+(n-1)$? Perhaps you meant to require $a,b>1$ but you should specify that.
– lulu
5 hours ago






1




1




Dirichlet’s theorem is useless in that kind of situation; the problem is much, much simpler. Aside from the obvious case $n=(n-1)+1$, you can try to find general patterns when $n=4m$, $n=2m+1$, $n=4m+2$.
– Mindlack
5 hours ago




Dirichlet’s theorem is useless in that kind of situation; the problem is much, much simpler. Aside from the obvious case $n=(n-1)+1$, you can try to find general patterns when $n=4m$, $n=2m+1$, $n=4m+2$.
– Mindlack
5 hours ago




1




1




True i should have specified that a,b>1, sorry, my bad.
– Daniel Gimpelman
5 hours ago




True i should have specified that a,b>1, sorry, my bad.
– Daniel Gimpelman
5 hours ago












Daniel, you can edit your post. Also, have a look at this introduction to posting mathematical notation.
– hardmath
5 hours ago




Daniel, you can edit your post. Also, have a look at this introduction to posting mathematical notation.
– hardmath
5 hours ago












Mindlack thanks, but I don't really understand how to go from here. How do I represent a and b even in a specific case? $7=2*3+1$ where $a=2$ and $b=1$? $8=4*2$ where $a=4$ and $b=0$? (And than b isn't co-prime). $10=4*2+2$ where $a=4$ and $b=2$ (a,b aren't co-prime). Also there would always be the m, which i'm not sure if the m could be greater than 1.
– Daniel Gimpelman
4 hours ago






Mindlack thanks, but I don't really understand how to go from here. How do I represent a and b even in a specific case? $7=2*3+1$ where $a=2$ and $b=1$? $8=4*2$ where $a=4$ and $b=0$? (And than b isn't co-prime). $10=4*2+2$ where $a=4$ and $b=2$ (a,b aren't co-prime). Also there would always be the m, which i'm not sure if the m could be greater than 1.
– Daniel Gimpelman
4 hours ago












2 Answers
2






active

oldest

votes

















up vote
5
down vote













Just to provide an answer synthesized out of the comments already posted, your best (read as easiest) approach to this kind of problem is to toy around with general patterns until something either clicks and you can write a clever proof or until you accidentally exhaust all possible cases.



In this particular problem, we can break down cases into the residue classes $bmod 4$ in order to hunt for patterns:



1) If $n=2k+1$ then the decomposition $n=(k)+(k+1)$ satisfies our criterion since consecutive numbers are always coprime and $kgeq 3$.



2) If $n=4k$ then consider the decomposition $n=(2k-1)+(2k+1)$. Are these numbers coprime? We can no longer rely upon the general fact that consecutive numbers are coprime, since these are not consecutive. However, if two numbers differ by exactly $2$, what is the only prime factor that they can share? In general, if two numbers differ by $m$, what prime factors can they share? Finally, are we sure that these numbers are both greater than $1$?



I have basically given away the entire answer, but I didn't know how to discuss this phenomenon in any other way, so I leave the final details of the second case, and the entirety of the third case, to you.






share|cite|improve this answer








New contributor




RandomMathDude is a new contributor to this site. Take care in asking for clarification, commenting, and answering.
Check out our Code of Conduct.














  • 2




    +1 A nice combination of hint and solution.
    – Ethan Bolker
    4 hours ago


















up vote
0
down vote













A different emphasis: if Euler's totient $phi(n) geq 3,$ then there is some integer $a$ with $gcd(a,n) = 1$ and $1 < a < n-1.$ If we then name $b = n-a,$ we find that $gcd(a,b) = 1$ as well, since a prime $p$ that divides both $a,n-a$ also divides $n,$ and this contradicts $gcd(a,n) = 1.$



So, when is $phi(n) geq 3 ; ? ; ;$ If $n$ is divisible by any prime $q geq 5,$ then $phi(n)$ is a multiple of $phi(q) = q-1,$ and that is at least $4.$



Next, if $n = 2^c ; 3^d ; . ;$ When $d=0$ we find $phi(n) = 2^{c-1}$ is at least $3$ when $c geq 3,$ leaving $2,4$ out. When $c=0$ we find $phi(n) = 2 cdot 3^{d-1}$ is at least $3$ when $d geq 2,$ leaving $3$ out. When $c,d geq 1,$ we find $phi(n) = 2^c cdot 3^{d-1}$ is at least $3$ when either $c geq 2$ or $d geq 2,$ so this leaves out $6.$



Put it together, for $n=5$ or $n geq 7,$ there is some $a$ with $1 < a < n-1$ and $gcd(a,n) = 1.$






share|cite|improve this answer





















  • Note that I hinted at this method a couple hours ago in the comments on the question. I purposely avoided taking it to a full solution in the hope that doing so would yield a better learning experience. Perhaps you might consider hiding some of the above in spoilers to help in that regard.
    – Bill Dubuque
    2 hours ago













Your Answer





StackExchange.ifUsing("editor", function () {
return StackExchange.using("mathjaxEditing", function () {
StackExchange.MarkdownEditor.creationCallbacks.add(function (editor, postfix) {
StackExchange.mathjaxEditing.prepareWmdForMathJax(editor, postfix, [["$", "$"], ["\\(","\\)"]]);
});
});
}, "mathjax-editing");

StackExchange.ready(function() {
var channelOptions = {
tags: "".split(" "),
id: "69"
};
initTagRenderer("".split(" "), "".split(" "), channelOptions);

StackExchange.using("externalEditor", function() {
// Have to fire editor after snippets, if snippets enabled
if (StackExchange.settings.snippets.snippetsEnabled) {
StackExchange.using("snippets", function() {
createEditor();
});
}
else {
createEditor();
}
});

function createEditor() {
StackExchange.prepareEditor({
heartbeatType: 'answer',
convertImagesToLinks: true,
noModals: true,
showLowRepImageUploadWarning: true,
reputationToPostImages: 10,
bindNavPrevention: true,
postfix: "",
imageUploader: {
brandingHtml: "Powered by u003ca class="icon-imgur-white" href="https://imgur.com/"u003eu003c/au003e",
contentPolicyHtml: "User contributions licensed under u003ca href="https://creativecommons.org/licenses/by-sa/3.0/"u003ecc by-sa 3.0 with attribution requiredu003c/au003e u003ca href="https://stackoverflow.com/legal/content-policy"u003e(content policy)u003c/au003e",
allowUrls: true
},
noCode: true, onDemand: true,
discardSelector: ".discard-answer"
,immediatelyShowMarkdownHelp:true
});


}
});






Daniel Gimpelman is a new contributor. Be nice, and check out our Code of Conduct.










draft saved

draft discarded


















StackExchange.ready(
function () {
StackExchange.openid.initPostLogin('.new-post-login', 'https%3a%2f%2fmath.stackexchange.com%2fquestions%2f3041656%2fsum-of-two-co-prime-integers%23new-answer', 'question_page');
}
);

Post as a guest















Required, but never shown

























2 Answers
2






active

oldest

votes








2 Answers
2






active

oldest

votes









active

oldest

votes






active

oldest

votes








up vote
5
down vote













Just to provide an answer synthesized out of the comments already posted, your best (read as easiest) approach to this kind of problem is to toy around with general patterns until something either clicks and you can write a clever proof or until you accidentally exhaust all possible cases.



In this particular problem, we can break down cases into the residue classes $bmod 4$ in order to hunt for patterns:



1) If $n=2k+1$ then the decomposition $n=(k)+(k+1)$ satisfies our criterion since consecutive numbers are always coprime and $kgeq 3$.



2) If $n=4k$ then consider the decomposition $n=(2k-1)+(2k+1)$. Are these numbers coprime? We can no longer rely upon the general fact that consecutive numbers are coprime, since these are not consecutive. However, if two numbers differ by exactly $2$, what is the only prime factor that they can share? In general, if two numbers differ by $m$, what prime factors can they share? Finally, are we sure that these numbers are both greater than $1$?



I have basically given away the entire answer, but I didn't know how to discuss this phenomenon in any other way, so I leave the final details of the second case, and the entirety of the third case, to you.






share|cite|improve this answer








New contributor




RandomMathDude is a new contributor to this site. Take care in asking for clarification, commenting, and answering.
Check out our Code of Conduct.














  • 2




    +1 A nice combination of hint and solution.
    – Ethan Bolker
    4 hours ago















up vote
5
down vote













Just to provide an answer synthesized out of the comments already posted, your best (read as easiest) approach to this kind of problem is to toy around with general patterns until something either clicks and you can write a clever proof or until you accidentally exhaust all possible cases.



In this particular problem, we can break down cases into the residue classes $bmod 4$ in order to hunt for patterns:



1) If $n=2k+1$ then the decomposition $n=(k)+(k+1)$ satisfies our criterion since consecutive numbers are always coprime and $kgeq 3$.



2) If $n=4k$ then consider the decomposition $n=(2k-1)+(2k+1)$. Are these numbers coprime? We can no longer rely upon the general fact that consecutive numbers are coprime, since these are not consecutive. However, if two numbers differ by exactly $2$, what is the only prime factor that they can share? In general, if two numbers differ by $m$, what prime factors can they share? Finally, are we sure that these numbers are both greater than $1$?



I have basically given away the entire answer, but I didn't know how to discuss this phenomenon in any other way, so I leave the final details of the second case, and the entirety of the third case, to you.






share|cite|improve this answer








New contributor




RandomMathDude is a new contributor to this site. Take care in asking for clarification, commenting, and answering.
Check out our Code of Conduct.














  • 2




    +1 A nice combination of hint and solution.
    – Ethan Bolker
    4 hours ago













up vote
5
down vote










up vote
5
down vote









Just to provide an answer synthesized out of the comments already posted, your best (read as easiest) approach to this kind of problem is to toy around with general patterns until something either clicks and you can write a clever proof or until you accidentally exhaust all possible cases.



In this particular problem, we can break down cases into the residue classes $bmod 4$ in order to hunt for patterns:



1) If $n=2k+1$ then the decomposition $n=(k)+(k+1)$ satisfies our criterion since consecutive numbers are always coprime and $kgeq 3$.



2) If $n=4k$ then consider the decomposition $n=(2k-1)+(2k+1)$. Are these numbers coprime? We can no longer rely upon the general fact that consecutive numbers are coprime, since these are not consecutive. However, if two numbers differ by exactly $2$, what is the only prime factor that they can share? In general, if two numbers differ by $m$, what prime factors can they share? Finally, are we sure that these numbers are both greater than $1$?



I have basically given away the entire answer, but I didn't know how to discuss this phenomenon in any other way, so I leave the final details of the second case, and the entirety of the third case, to you.






share|cite|improve this answer








New contributor




RandomMathDude is a new contributor to this site. Take care in asking for clarification, commenting, and answering.
Check out our Code of Conduct.









Just to provide an answer synthesized out of the comments already posted, your best (read as easiest) approach to this kind of problem is to toy around with general patterns until something either clicks and you can write a clever proof or until you accidentally exhaust all possible cases.



In this particular problem, we can break down cases into the residue classes $bmod 4$ in order to hunt for patterns:



1) If $n=2k+1$ then the decomposition $n=(k)+(k+1)$ satisfies our criterion since consecutive numbers are always coprime and $kgeq 3$.



2) If $n=4k$ then consider the decomposition $n=(2k-1)+(2k+1)$. Are these numbers coprime? We can no longer rely upon the general fact that consecutive numbers are coprime, since these are not consecutive. However, if two numbers differ by exactly $2$, what is the only prime factor that they can share? In general, if two numbers differ by $m$, what prime factors can they share? Finally, are we sure that these numbers are both greater than $1$?



I have basically given away the entire answer, but I didn't know how to discuss this phenomenon in any other way, so I leave the final details of the second case, and the entirety of the third case, to you.







share|cite|improve this answer








New contributor




RandomMathDude is a new contributor to this site. Take care in asking for clarification, commenting, and answering.
Check out our Code of Conduct.









share|cite|improve this answer



share|cite|improve this answer






New contributor




RandomMathDude is a new contributor to this site. Take care in asking for clarification, commenting, and answering.
Check out our Code of Conduct.









answered 4 hours ago









RandomMathDude

1512




1512




New contributor




RandomMathDude is a new contributor to this site. Take care in asking for clarification, commenting, and answering.
Check out our Code of Conduct.





New contributor





RandomMathDude is a new contributor to this site. Take care in asking for clarification, commenting, and answering.
Check out our Code of Conduct.






RandomMathDude is a new contributor to this site. Take care in asking for clarification, commenting, and answering.
Check out our Code of Conduct.








  • 2




    +1 A nice combination of hint and solution.
    – Ethan Bolker
    4 hours ago














  • 2




    +1 A nice combination of hint and solution.
    – Ethan Bolker
    4 hours ago








2




2




+1 A nice combination of hint and solution.
– Ethan Bolker
4 hours ago




+1 A nice combination of hint and solution.
– Ethan Bolker
4 hours ago










up vote
0
down vote













A different emphasis: if Euler's totient $phi(n) geq 3,$ then there is some integer $a$ with $gcd(a,n) = 1$ and $1 < a < n-1.$ If we then name $b = n-a,$ we find that $gcd(a,b) = 1$ as well, since a prime $p$ that divides both $a,n-a$ also divides $n,$ and this contradicts $gcd(a,n) = 1.$



So, when is $phi(n) geq 3 ; ? ; ;$ If $n$ is divisible by any prime $q geq 5,$ then $phi(n)$ is a multiple of $phi(q) = q-1,$ and that is at least $4.$



Next, if $n = 2^c ; 3^d ; . ;$ When $d=0$ we find $phi(n) = 2^{c-1}$ is at least $3$ when $c geq 3,$ leaving $2,4$ out. When $c=0$ we find $phi(n) = 2 cdot 3^{d-1}$ is at least $3$ when $d geq 2,$ leaving $3$ out. When $c,d geq 1,$ we find $phi(n) = 2^c cdot 3^{d-1}$ is at least $3$ when either $c geq 2$ or $d geq 2,$ so this leaves out $6.$



Put it together, for $n=5$ or $n geq 7,$ there is some $a$ with $1 < a < n-1$ and $gcd(a,n) = 1.$






share|cite|improve this answer





















  • Note that I hinted at this method a couple hours ago in the comments on the question. I purposely avoided taking it to a full solution in the hope that doing so would yield a better learning experience. Perhaps you might consider hiding some of the above in spoilers to help in that regard.
    – Bill Dubuque
    2 hours ago

















up vote
0
down vote













A different emphasis: if Euler's totient $phi(n) geq 3,$ then there is some integer $a$ with $gcd(a,n) = 1$ and $1 < a < n-1.$ If we then name $b = n-a,$ we find that $gcd(a,b) = 1$ as well, since a prime $p$ that divides both $a,n-a$ also divides $n,$ and this contradicts $gcd(a,n) = 1.$



So, when is $phi(n) geq 3 ; ? ; ;$ If $n$ is divisible by any prime $q geq 5,$ then $phi(n)$ is a multiple of $phi(q) = q-1,$ and that is at least $4.$



Next, if $n = 2^c ; 3^d ; . ;$ When $d=0$ we find $phi(n) = 2^{c-1}$ is at least $3$ when $c geq 3,$ leaving $2,4$ out. When $c=0$ we find $phi(n) = 2 cdot 3^{d-1}$ is at least $3$ when $d geq 2,$ leaving $3$ out. When $c,d geq 1,$ we find $phi(n) = 2^c cdot 3^{d-1}$ is at least $3$ when either $c geq 2$ or $d geq 2,$ so this leaves out $6.$



Put it together, for $n=5$ or $n geq 7,$ there is some $a$ with $1 < a < n-1$ and $gcd(a,n) = 1.$






share|cite|improve this answer





















  • Note that I hinted at this method a couple hours ago in the comments on the question. I purposely avoided taking it to a full solution in the hope that doing so would yield a better learning experience. Perhaps you might consider hiding some of the above in spoilers to help in that regard.
    – Bill Dubuque
    2 hours ago















up vote
0
down vote










up vote
0
down vote









A different emphasis: if Euler's totient $phi(n) geq 3,$ then there is some integer $a$ with $gcd(a,n) = 1$ and $1 < a < n-1.$ If we then name $b = n-a,$ we find that $gcd(a,b) = 1$ as well, since a prime $p$ that divides both $a,n-a$ also divides $n,$ and this contradicts $gcd(a,n) = 1.$



So, when is $phi(n) geq 3 ; ? ; ;$ If $n$ is divisible by any prime $q geq 5,$ then $phi(n)$ is a multiple of $phi(q) = q-1,$ and that is at least $4.$



Next, if $n = 2^c ; 3^d ; . ;$ When $d=0$ we find $phi(n) = 2^{c-1}$ is at least $3$ when $c geq 3,$ leaving $2,4$ out. When $c=0$ we find $phi(n) = 2 cdot 3^{d-1}$ is at least $3$ when $d geq 2,$ leaving $3$ out. When $c,d geq 1,$ we find $phi(n) = 2^c cdot 3^{d-1}$ is at least $3$ when either $c geq 2$ or $d geq 2,$ so this leaves out $6.$



Put it together, for $n=5$ or $n geq 7,$ there is some $a$ with $1 < a < n-1$ and $gcd(a,n) = 1.$






share|cite|improve this answer












A different emphasis: if Euler's totient $phi(n) geq 3,$ then there is some integer $a$ with $gcd(a,n) = 1$ and $1 < a < n-1.$ If we then name $b = n-a,$ we find that $gcd(a,b) = 1$ as well, since a prime $p$ that divides both $a,n-a$ also divides $n,$ and this contradicts $gcd(a,n) = 1.$



So, when is $phi(n) geq 3 ; ? ; ;$ If $n$ is divisible by any prime $q geq 5,$ then $phi(n)$ is a multiple of $phi(q) = q-1,$ and that is at least $4.$



Next, if $n = 2^c ; 3^d ; . ;$ When $d=0$ we find $phi(n) = 2^{c-1}$ is at least $3$ when $c geq 3,$ leaving $2,4$ out. When $c=0$ we find $phi(n) = 2 cdot 3^{d-1}$ is at least $3$ when $d geq 2,$ leaving $3$ out. When $c,d geq 1,$ we find $phi(n) = 2^c cdot 3^{d-1}$ is at least $3$ when either $c geq 2$ or $d geq 2,$ so this leaves out $6.$



Put it together, for $n=5$ or $n geq 7,$ there is some $a$ with $1 < a < n-1$ and $gcd(a,n) = 1.$







share|cite|improve this answer












share|cite|improve this answer



share|cite|improve this answer










answered 2 hours ago









Will Jagy

101k598198




101k598198












  • Note that I hinted at this method a couple hours ago in the comments on the question. I purposely avoided taking it to a full solution in the hope that doing so would yield a better learning experience. Perhaps you might consider hiding some of the above in spoilers to help in that regard.
    – Bill Dubuque
    2 hours ago




















  • Note that I hinted at this method a couple hours ago in the comments on the question. I purposely avoided taking it to a full solution in the hope that doing so would yield a better learning experience. Perhaps you might consider hiding some of the above in spoilers to help in that regard.
    – Bill Dubuque
    2 hours ago


















Note that I hinted at this method a couple hours ago in the comments on the question. I purposely avoided taking it to a full solution in the hope that doing so would yield a better learning experience. Perhaps you might consider hiding some of the above in spoilers to help in that regard.
– Bill Dubuque
2 hours ago






Note that I hinted at this method a couple hours ago in the comments on the question. I purposely avoided taking it to a full solution in the hope that doing so would yield a better learning experience. Perhaps you might consider hiding some of the above in spoilers to help in that regard.
– Bill Dubuque
2 hours ago












Daniel Gimpelman is a new contributor. Be nice, and check out our Code of Conduct.










draft saved

draft discarded


















Daniel Gimpelman is a new contributor. Be nice, and check out our Code of Conduct.













Daniel Gimpelman is a new contributor. Be nice, and check out our Code of Conduct.












Daniel Gimpelman is a new contributor. Be nice, and check out our Code of Conduct.
















Thanks for contributing an answer to Mathematics Stack Exchange!


  • Please be sure to answer the question. Provide details and share your research!

But avoid



  • Asking for help, clarification, or responding to other answers.

  • Making statements based on opinion; back them up with references or personal experience.


Use MathJax to format equations. MathJax reference.


To learn more, see our tips on writing great answers.





Some of your past answers have not been well-received, and you're in danger of being blocked from answering.


Please pay close attention to the following guidance:


  • Please be sure to answer the question. Provide details and share your research!

But avoid



  • Asking for help, clarification, or responding to other answers.

  • Making statements based on opinion; back them up with references or personal experience.


To learn more, see our tips on writing great answers.




draft saved


draft discarded














StackExchange.ready(
function () {
StackExchange.openid.initPostLogin('.new-post-login', 'https%3a%2f%2fmath.stackexchange.com%2fquestions%2f3041656%2fsum-of-two-co-prime-integers%23new-answer', 'question_page');
}
);

Post as a guest















Required, but never shown





















































Required, but never shown














Required, but never shown












Required, but never shown







Required, but never shown

































Required, but never shown














Required, but never shown












Required, but never shown







Required, but never shown







Popular posts from this blog

Quarter-circle Tiles

build a pushdown automaton that recognizes the reverse language of a given pushdown automaton?

Mont Emei